LSAT and Law School Admissions Forum

Get expert LSAT preparation and law school admissions advice from PowerScore Test Preparation.

 jgray
  • Posts: 41
  • Joined: Feb 13, 2015
|
#32339
Question asks for what the author would least likely to agree with. The explanation in the book is a little thin. Is D correct b/c it tries to compare the given restrictive policies to an un-discussed liberal one?
 Adam Tyson
PowerScore Staff
  • PowerScore Staff
  • Posts: 5153
  • Joined: Apr 14, 2011
|
#32354
That's my take on it, jgray, in addition to that answer being so broad, so general. The passage doesn't do anything to expand these ideas beyond the Cultural Revolution in China, but limits itself to that one time and place and what happened then and there.

Was there another answer that you liked better? For these "least" or "except" questions, typically what you are looking for is support for the wrong answers, rather than trying to show why one answer is correct. If you find support for it, it's out, and if you don't, it's the winner. You should have found more and better support for all the other answers here.

Take that approach, focused on eliminating wrong answers, and you won't need to worry too much about why the best answer is the best answer. It's best simply by virtue of the others being eliminated!
 mshaikh
  • Posts: 36
  • Joined: Jun 11, 2017
|
#36926
For this question, I was stuck between answer choice b and d because I felt that both were unsupported by the passage. How do you know to choose answer choice d over answer choice b? What lines/sentences provide support for answer choice b?
 nicholaspavic
PowerScore Staff
  • PowerScore Staff
  • Posts: 271
  • Joined: Jun 12, 2017
|
#37253
Hi ms shaikh,
Good question. This kind of Cannot Be True question from the author's perspective can be a bit tricky because many of the incorrect answers will only be partially supportable from the passage. In other words, the testmaker will play with the degree of uncertainty. Also, always keep in mind what you are really looking for in the right answers, always being careful not to get turned around! Here Answer option (B) is partially supportable from the sentence beginning at Line 15 of the passage:

"Ironically, the same set of requirements that constricted artistic expression during the Cultural Revolution has had the opposite effect since; many artistic movements have flourished in reaction to the (20) monotony of Revolutionary Realism."

That line of "many artistic movements" makes answer choice (B) partially supported and therefore incorrect.

Thanks for the great question! :-D
 mshaikh
  • Posts: 36
  • Joined: Jun 11, 2017
|
#37347
Hi NicholasPavic ,

Thanks so much for this explanation and a few others as well! This makes much more sense now. :-D
 kwcflynn
  • Posts: 41
  • Joined: Nov 25, 2018
|
#62746
Hi!

Could it be argued that: (B) is the credited credit answer because the author's neutral viewpoint does expose any interest or fascination with any art movement that developed after the Cultural Revolution and (D) is overly broad?

THANK YOU.
 Jay Donnell
PowerScore Staff
  • PowerScore Staff
  • Posts: 144
  • Joined: Jan 09, 2019
|
#62748
Hi kwcflyyn!

The purpose of number nine is to discover what the author is LEAST likely to agree with, which means that the correct response must be something that can clearly be seen to be in conflict with the author's position.

The author's largest contribution to tone/attitude came in the sentence that read: "Ironically, the same set of requirements that constricted artistic expression during the Cultural Revolution has had the opposite effect since; many artistic movements have flourished in reaction to the monotony of Revolutionary Realism."

If the author found it ironic that the restrictive policies in the Cultural Revolution would lead to a flourish in creative movements, then they could easily be said to disagree with the idea in D that: Its effects demonstrate that restrictive policies generally foster artistic growth more than liberal policies do." If these policies generally led to growth, the author would not have called this phenomenon ironic.

I think the point can be argued that the author would actually be in reasonable agreement with the claim in answer choice B, as the author clearly wasn't fond of the limited freedom of expression afforded by Maoist Revolutionary Realism as described as:
"dictated the content and style of Chinese art even before 1966 by requiring that artists "truthfully" depict the realities of socialist life in China. Interest in nonsocial, nonpolitical subjects was strictly forbidden, and, during the Cultural Revolution, what constituted truth was entirely for revolutionary forces to decide--the only reality artists could portray was one that had been thoroughly colored and distorted by political ideology."


I hope that helps to clear up the difference between the two answers and to help further illuminate why D wins!

Get the most out of your LSAT Prep Plus subscription.

Analyze and track your performance with our Testing and Analytics Package.